OBSTETRICS

 Preconception Care

Preconception care is intended to prevent congenital anomalies, maximize maternal health and is offered to all women of childbearing age, since more than 50% of pregnancies are unplanned.

  1. Proven benefits. Prevention of NTDs by folic acid supplementation. Tight glucose control in DM results in lower incidence of congenital abnormalities. Identify and when possible control factors leading to previous poor results in pregnancy (such as preterm labor from bacterial vaginosis, antiphospholipid antibody syndrome, etc.).
  2. Pre-pregnancy advice. Proper nutrition, exercise, smoking cessation, abstinence from alcohol and drugs, protection from radiation (x-rays) and workplace exposures, information on prescribed and OTC drugs to avoid teratogenicity, infection control (STD protection and treatment, rubella and hepatitis immunity status), and psychosocial counseling for planning a pregnancy. Assess risk for congenital disease (thalassemia, Tay-Sachs, cystic fibrosis, sickle cell, etc.).

Obstetrics: Prenatal Care

  1. History at Initial Evaluation.
    1. Menstrual history. Cycle length, age of menarche, pain with menses, duration of flow, characteristics of previous two menses, previous methods of contraception. Establish dates carefully based on first day of last menstrual period and uterine size. Obtain ultrasound if in doubt. Ultrasound dating is most accurate early in the pregnancy.
    2. Medical history. Underlying problems or illnesses, history of sexually transmitted diseases, medications, family history, and genetic history.
    3. Habits. Tobacco, alcohol, other recreational drugs, diet, activity, caffeine.
    4. Obstetric history. Dates of all pregnancies including terminations and spontaneous abortions. Outcome and gestational length. Duration and complications of labor. Particular note should be made of previous shoulder dystocia, premature labor, premature rupture of membranes (PROM), placenta previa, and postpartum hemorrhage. The type of previous deliveries is also important: normal spontaneous vaginal delivery (NSVD), forceps, C-section (indication, type of uterine incision). Weight, sex, and Apgar scores of liveborn infants. Neonatal complications. Number of living children.
    5. Social history. Occupational hazards, support network, whether or not the father of child is involved, wanted or unwanted pregnancy, expectations, potential stresses, need of social or financial services. Domestic violence or sexual assault. See below.
    6. Prenatal Care for Patients at Risk for Preterm Labor. Frequent visits during weeks 22 to 32, cervical group B streptococci and urine culture at 24 weeks, vaginal exam for pH and cervical exam, monitor uterine activity, education on nutrition and preterm labor, and reinforce what signs and symptoms to watch for (abdominal cramping, pressure, cramps, backache, increased vaginal discharge, fluid leak, regular uterine contractions).
  2. Physical Exam.
    1. General physical exam. Particular attention to height, weight, BP, thyroid gland, dentition, heart, breasts, deep tendon reflexes, signs of underlying heart disease.
    2. Pelvic examination.
      1. External. Look for evidence of condylomata acuminata. These lesions may progress during pregnancy, and a small percentage of infants born through involved vaginal tissue will develop laryngeal papillomas or anogenital warts. Podophyllin is contraindicated during pregnancy, but cryotherapy, laser, and TCA may be used. Also look for and culture lesions suspicious for herpes simplex.
      2. Vaginal and cervical. Look for evidence of condylomas and herpes. Examine vaginal discharge and evaluate for Candida, Trichomonas, and bacterial vaginosis (BV); culture cervical discharge for GC and Chlamydia. Treat any vaginal infection. It is particularly important to screen for and treat BV, since it is associated with an increased risk of preterm labor, premature rupture of membranes, preterm birth, and histologic chorioamnionitis. (See Chapter 13 for treatment.) Rule out cervical anomalies. Pap smear should be obtained if patient has not had one in last 6 months.
      3. Bimanual. Rule out adnexal abnormalities. Determine uterine size: 8 weeks = 2 x normal; 10 weeks = 3 x normal; 12 weeks = 4 x normal; 16 weeks = halfway to umbilicus; 20 weeks = at umbilicus; fundal height (weeks of gestation = cm from pubic symphysis to fundus).
  3. Laboratory Evaluation at first prenatal visit.
    1. Routine. Pap smear, CBC, UA, and culture to screen for bacteriuria, ABO blood type, Rh type, antibody screen (indirect Coombs’), VDRL test, rubella antibody titer, and hepatitis B surface antigen. Treat asymptomatic bacteriuria to prevent pyelonephritis during pregnancy. Urine should also be screened for protein and glucose by dipstick at each visit. Offer HIV testing. (Many states require testing since AZT administered during pregnancy substantially decreases risk of transmission of HIV to infant&endash;see Chapter 11.)
    2. When indicated. Cervical culture for GC and Chlamydia, Toxoplasmosis antibody test, sickle cell preparation, or hemoglobin electrophoresis in all previously unscreened black women, tuberculin skin testing, HIV antibody testing, and CMV titers.
  4. Labs During Pregnancy.
    1. Every visit. Dipstick urine for protein and glucose.
    2. 15 to 20 weeks. Serum triple-screen (alpha-fetoprotein [AFP], beta-HCG, and estradiol). See below.
    3. 14 to 20 weeks. Amniocentesis, when indicated.
    4. 24 to 28 weeks. Blood glucose screen after 50 g of oral glucose and urine culture.
    5. 28 to 32 weeks. Hematocrit
    6. 36 weeks. Rh antibody screening if indicated. Consider GC, Chlamydia, and herpes rescreening in high-risk women. Repeat hematocrit if indicated. Consider testing for group B streptococci at 35 to 37 weeks (see recommendations below). Consider starting acyclovir for patients with genital HSV which has been shown to decrease transmission to the infant if given prophylactically.
  5. Expected Weight Gain.
    1. First trimester. Should gain 2 to 5 lb total.
    2. After first trimester. 3/4 to 1 lb per week.
    3. Average total weight gain. 25 ± 5 lb.

Obstetrics: Prenatal Patient Education

  1. Nutrition in Pregnancy.
    1. Caloric requirements. 30 to 35 kcal/kg/day plus 300 kcal/day. Requirements are higher in adolescence and with multiple gestation.
    2. Calcium.
      1. Requirements. 1200 to 1500 mg of elemental calcium per day.
      2. Calcium supplements. Milk, 8 oz glass: 300 mg of calcium, yogurt; generic calcium carbonate (260 mg Ca/650 mg tablet); calcium gluconate chewable (45 mg Ca/500 mg tablets; Tums regular strength (200 mg Ca/500 mg chewable tablet).
    3. Iron.
      1. Requirements. 30 mg of elemental iron per day.
      2. Additional requirements. If the pregnant woman is iron deficient or has a multiple-gestation pregnancy, she should take 60 to 100 mg of elemental iron per day. If her Hb is <10, she requires 200 mg/day.
      3. Iron supplements.
        1. Ferrous sulfate 65 mg of elemental Fe per 324 mg tablet (20% elemental iron).
        2. Ferrous fumarate 106 mg Fe/325 mg tablet (33% elemental iron).
        3. Ferrous gluconate 38 mg Fe/325 mg (11.6% elemental iron).
    4. Folic acid.
      1. Requirements. 0.4 mg to 1 mg/day. Most prescription prenatal vitamins contain 1 mg of folate. OTC prenatal vitamins contain 0.8 mg of folic acid.
      2. Sources. Green leafy vegetables, broccoli, mushrooms, and liver.
      3. Adequate folate before conception has been shown to reduce the risk of NTDs.
      4. Prior history of NTD or family history of NTD&endash;recommend 4 mg /day for 1 month before pregnancy and during first 3 months gestation.
  2. Activity.
    1. Occupation. Abstinence from physical work may be recommended if the woman has a history of two previous premature deliveries, an incompetent cervix, or fetal loss secondary to uterine anomalies. No controlled clinical trials have demonstrated efficacy of bed rest for any of these conditions. Women with premature rupture of membranes (PROM), CHF, hemoglobinopathies, Marfan’s syndrome, or diabetes with multiple end-organ involvement are also at risk for complications and may benefit from reduced activity as can women with multiple gestations at over 28 weeks. Bed rest is indicated if there is a suspicion of IUGR, preeclampsia, or preterm labor.
    2. Exercise. (Expert opinion not controlled studies) Avoid in supine position after 1st trimester. Avoid exercise with potential for abdominal trauma. Exercise to fatigue not exhaustion. Contraindicated if pregnancy induced hypertension (PIH), PROM, PTL (preterm labor), incompetent cervix, vaginal bleeding, IUGR.
    3. Other. There are no routine restrictions on sexual relations, other than comfort and position. Caution should be used if any of the conditions listed above apply.
  3. Habits and Miscellaneous.
    1. Alcohol. Increases the risk of midtrimester abortion, mental retardation, behavior and learning disorders. 10% to 30% risk of fetal alcohol syndrome in offspring of women who drink 3 to 5 drinks per day. Risks with lesser consumption unknown.
    2. Tobacco. Increases the risk of low-birth-weight infants, premature labor, spontaneous abortions, stillbirth, and birth defects.
    3. Crack cocaine or other illicit drug use. Associated with perinatal addiction, preterm labor, and cognitive and psychologic difficulties in the infant. Cocaine abuse during pregnancy is associated with a significant increase in the incidence of placental abruption.
    4. Caffeine. 100 mg/day or 1 cup of coffee (approximately 1 cup of coffee per day) does not increase the risk of spontaneous abortions or IUGR.
    5. Seatbelts. A seatbelt should be worn such that the belts do not directly cross the gravid uterus; lap belt low over hips/ shoulder belt above uterus.
    6. Medications. In general, no medications should be used without checking with a physician.
      1. FDA classification of medication with regard to adverse fetal effects. Category A: proved safe for fetus in human studies (such as prenatal vitamins); Category B: adverse effect not demonstrated in animal studies with no human studies, or adverse effects shown in animal studies have not been reproduced in human studies (as with penicillin); Category C: no adequate animal or human studies are available, or animal studies show adverse fetal effects with no human data; Category D: evidence of fetal risk but benefits believed to outweigh the risks (as with carbamazepine); Category X: drugs with proved fetal risks that outweigh any benefits.
      2. It is recommended that you confirm the category of all medications in pregnancy before prescribing or recommending them to your patient.
      3. Drugs that are used in pregnancy with no known adverse effect at the usual dose (some are class B). Antihistamines, decongestants (e.g., pseudoephedrine), some antibiotics, (penicillin, ampicillin, cephalosporins, erythromycin), non-quinine antimalarials, general anesthetics, acetaminophen, tuberculostatics (INH, PAS, and rif-ampin), metronidazole (avoid in first trimester if possible, though one study showed no teratogenicity), steroids. Accidental use of clomiphene, bromocriptine, birth control pills, and vaginal spermicides have shown no adverse effects.
    7. Infections. Avoid children with viral illnesses, especially if not rubella or CMV immune. Avoid direct contact with cat litter and eating raw meat to minimize contact with Toxoplasma gondii (toxoplasmosis).
    8. Domestic Violence Screen all pregnant women for domestic violence. Violence often escalates with pregnancy. Affects 0.9%-20% of pregnant women in US. Ask direct questions, for example, "Have you been hit, pushed or kicked in the last year?"
    9. Potential problems. Advise patient to contact physician if she experiences vaginal bleeding, leakage of fluid, fever, persistent nausea or vomiting, burning on urination, severe abdominal pain, severe headache or visual disturbance, persistent RUQ pain, peripheral edema, decrease in fetal movement. (Generally after quickening, one should expect 4 or more fetal movements per hour or at least 10 discrete movements in 2 hours.)

Obstetrics: Rh Screening and Rho(D) Immunoglobulin

  1. Protocol for Routine Rh Screening and Administration of Rho(D) Immunoglobulin.
    1. Initial visit. Draw blood for ABO group, Rh type, and antibody screening (indirect Coombs’).
    2. If patient is Rh negative. Repeat antibody screen at 26 weeks and, if no antibody is detected, give 300 mg of Rho(D) immunoglobulin (1 vial 5300 mg) IM. Can give Rho(D) immunoglobulin before knowing antibody result. If antibody is detected, see "additional Rho(D)" below.
    3. After delivery. Check fetal ABO/Rh type. If infant is Rh positive, mother receives 300 mg of Rho(D) immunoglobulin IM within 72 hours of delivery.
    4. 1 vial suppresses immunity to approximately 30 ml of whole blood (15 ml of Rh(+) packed RBCs).
  2. Additional Rho(D) Immunoglobulin Requirements.
    1. If at anytime during pregnancy a fetal-maternal hemorrhage is suspected, a Kleihauer-Betke (acid elution) test should be performed. If positive, the mother should receive 10 mg of Rho(D) immunoglobulin per milliliter of fetal blood calculated to have entered the maternal circulation. However, the Kleihauer-Betke test is not 100% sensitive, and so if there is trauma and a suggestion of fetal-maternal hemorrhage, presumptive use of Rho (D) is indicated.
    2. A 50 mg dose of Rho (D) immunoglobulin (1 vial microdose = 50 mg) is indicated for an Rh-negative woman after a first trimester terminated or spontaneously aborted pregnancy.
    3. A 300 mg dose of Rho(D) immunoglobulin is indicated for the Rh- negative woman who undergoes amniocentesis, a spontaneous or induced abortion, or who has an ectopic pregnancy.
    4. The Kleihauer-Betke test should be performed after delivery if a larger than usual fetal-maternal hemorrhage may have taken place, as with placental abruption. More than the standard 300 mg dose may be required (which protects only up to 15 ml of Rh-positive red blood cells).
  3. Isoimmunization. If the patient is Rh negative and the antibody screen is positive before Rho(D) immunoglobulin administration, obtain an antibody titer and refer to a specialist. These infants are at risk for erythro-blastosis fetalis.

Obstetrics: Prenatal Diagnosis of Congenital Disorders

  1. Overview. Major congenital anomalies occur in 3% live born infants at term and represent the leading cause of infant mortality in the United States. Family history should be obtained to evaluate the risk of congenital disease. All women should be offered serum AFP/triple screen. A patient’s attitude toward termination should not influence the screening/counseling. Knowledge of fetal abnormality can facilitate psychological adjustment and in the aid care of the fetus.
  2. Methods of Screening/Diagnosis.
    1. AFP/Triple screen. Offer to all pregnant women at 15 to 20 weeks (see below).
    2. Chorionic villus sampling at 10 to 12 weeks (fetal loss rate of 0.5% to 1.5%). No longer associated with increase in limb defects.
    3. Early amniocentesis performed between 12 and 15 weeks with 1% to 2% fetal loss rate.
    4. Midtrimester amniocentesis between 15 and 20 weeks with 0.5% to 1% fetal loss rate.
    5. Fetal ultrasound

Obstetrics: Alpha-Fetoprotein (AFP)/Triple Screen

  1. Overview. The measurement of AFP, estriol, and HCG (triple screen) in maternal serum at 15 to 20 weeks of gestation is used as a screening test for fetal structural abnormalities and chromosomal abnormalities (trisomy 21). All three levels are dependent on maternal weight and gestational age. Since proper interpretation of AFP/HCG/estriol depends on fetal age, women with abnormal values should be referred for ultrasound to confirm gestational age and to evaluate for neural tube defects (NTD) and other structural abnormalities. In the United States, the incidence of NTD is roughly 1 per 1000 live births.
  2. If Ultrasound Dating confirms the patient’s dates (i.e., the triple screen is abnormal) but no diagnostic structural abnormalities are seen, the patient should be referred for amniocentesis.
  3. Risks. Psychologic stress, false positive results, false reassurance, and potential fetal trauma secondary to amniocentesis.
  4. Causes of an elevated AFP.
    1. Underestimated gestational age.
    2. Open NTDs (meningomyelocele, anencephaly).
    3. Fetal nephrosis and cystic hygroma.
    4. Fetal GI obstruction, omphalocele, gastroschisis.
    5. Prematurity, low birth weight, IUGR.
    6. Abdominal pregnancy.
    7. Multiple fetuses
    8. Fetal demise.
  5. Causes of a Low AFP.
    1. Overestimated gestational age.
    2. Missed abortions.
    3. Molar pregnancies.
    4. Chromosomal abnormalities (including Down syndrome).
  6. Causes of a Low Estriol, Elevated HCG, and Low AFP: Trisomy 21 (Down syndrome).

Obstetrics: Antenatal Fetal Surveillance

  1. Obstetric Ultrasound.
    1. Indications. Routine ultrasound is not indicated but can be used in the following situations:
      1. To determine the presence or absence of an intrauterine pregnancy.
      2. To determine gestational age.
      3. To measure fetal growth and identify intrauterine growth retardation.
      4. To identify multiple-gestation pregnancies.
      5. To detect fetal anomalies (nearly 100% sensitive for detection of NTD).
      6. To detect oligohydramnios or polyhydramnios (if size greater or less than dates).
      7. To demonstrate placental abnormalities (e.g., abruption, placenta previa).
      8. To identify maternal uterine and pelvic anomalies.
    2. Timing. Will depend on the indication for ultrasound.
      1. In general, the earlier ultrasound is performed in pregnancy, the more accurate is the EDC (first trimester ultrasound gives EDC +/- 5 days; second trimester EDC + 10 days; third trimester EDC +/- 3 weeks). Fetal anomalies may not become apparent until after 20 weeks.
      2. Amniotic fluid volume and fetal movement, tone, and breathing in conjunction with an NST can be used to calculate scores on biophysical profiles (BPP, see below). This can be helpful in the decision to induce or follow postdated pregnancies, high-risk pregnancies, or diabetic pregnancies.
  2. Nonstress Testing (NST).
    1. Timing. An NST should be done at the earliest point at which an intervention would be performed if a clearly abnormal result were obtained (generally 32 to 34 weeks).
    2. Indications. High-risk pregnancies including hypertension, diabetes mellitus, multiple gestation, suspected oligohydramnios or IUGR, known placental abnormality, maternal heart or renal disease, hemoglobinopathy, postdated pregnancies, previous unexplained fetal demise and maternal perceptions of decreased fetal movement.
    3. Equipment. External fetal heart rate monitor and uterine contraction monitor.
    4. Interpretation.
      1. A reassuring NST demonstrates three or more fetal movements accompanied by a fetal heart rate acceleration of 15 beats per minute or more lasting at least 15 seconds during a 20-minute period.
      2. Lack of fetal movement is non-diagnostic and a repeat NST should be performed after a meal. Lack of movement for short periods of time may be attributable to fetal sleep. However, absence of movement for prolonged periods of time may be ominous.
      3. The NST is abnormal when the criteria for a reassuring NST are not met or late or variable decelerations are present. A biophysical profile (BPP, below) is then indicated.
  3. Biophysical Profile (BPP).
    1. Indications. Same as for NST. May be used as early as 26 to 28 weeks for the surveillance of a complicated or high-risk pregnancy.
    2. Procedural details. Real-time ultrasound coupled with external fetal heart rate and uterine contraction monitoring.
    3. Interpretation. Five parameters are evaluated: Fetal breathing movements; gross body movements, fetal muscular tone, amniotic fluid volume (look for pocket of amniotic fluid that measures 2 cm in two perpendicular planes), reactivity of fetal heart rate.
      1. Each component of the BPP is given a score of 0 (parameter absent) or 2 (parameter present). The total score ranges from 0 (ominous) to 10 (reassuring; infant at low risk of asphyxia). Further discussion of this topic is beyond the scope of this chapter.
  4. Amniocentesis.
    1. Indications: Done at 14 to 18 weeks of gestation to identify selected inherited disorders in women at increased risk.
      1. Advanced maternal age (35 years of age or older).
      2. Previous pregnancy resulting in the birth of a child with a chromosomal abnormality.
      3. Down syndrome or other chromosome abnormality in either parent or close family member or if either parent is a carrier of a genetically transmitted disease (Duchenne’s muscular dystrophy, hemophilia, metabolic disease, etc.).
      4. NTD in either parent or a first-degree relative or previous child born with a NTD.
      5. Abnormal serum AFP/triple screen.
      6. To detect isoimmunization
      7. Determine fetal lung maturity.
        1. Lecithin-to-sphingomyelin (L/S) ratio. If L/S is >2.0, there is a low risk of respiratory distress secondary to prematurity.
        2. Phosphatidylglycerol (PG). PG first appears at 35 weeks gestation and increases in concentration until 40 weeks. If present, it provides reassurance of fetal lung maturity.

Obstetrics: Group B Streptococcal (GBS) Infection

  1. Risk factors for Neonatal Sepsis. Intrapartum chorioamnionitis, maternal group B streptococcal (GBS) colonization in the rectum or vagina, prolonged rupture of membranes, and prolonged monitoring with an internal pressure catheter or fetal scalp lead.
  2. Vertical Transmission of GBS. GBS is the number one cause of neonatal sepsis and meningitis in the United States. Infection occurs in 2 or 3 neonates per 1000 live births. Maternal colonization can be transient, and 20% to 25% of pregnant females are carriers at any given time. In addition to threatening the life of a neonate, GBS is also an important risk factor for the development of chorioamnionitis in the mother, thereby increasing morbidity and the rate of intrapartum complications.
  3. The CDC Recommends 2 Options for GBS.
    1. Option 1.
      1. Culture all women (rectal and vaginal) at 35-37 weeks. If the patient’s recto-vaginal cultures are positive for GBS, she should be offered intrapartum antibiotic prophylaxis.
      2. Treatment. Oral antibiotics are ineffective. The following regimens may be used:
        1. Penicillin G 5 million units IV and then 2.5 million units Q4h until delivery. Penicillin G is the preferred antibiotic because of its narrow spectrum, making it less likely to select for antibiotic-resistant bacteria.
        2. Ampicillin 2 g IV followed by 1 g Q4h until delivery.
        3. For penicillin allergy. Either clindamycin 900 mg IV Q8h or erythromycin 500 mg IV Q6h may be given until delivery.
    2. Option 2. Screening cultures are not done, but antibiotic prophylaxis is given if any of the following risk factors are present:
      1. Previously delivered neonate with GBS infection.
      2. GBS bacteriuria during the current pregnancy.
      3. Labor and delivery occur at less than 37 weeks of gestation (attack rates for preterm infants are higher).
      4. Membranes have been ruptured for >18 hours (12 hours in some institutions).
      5. Intrapartum temperature greater than or equal to 38.0° C (100.4° F).
      6. If PROM occurs at <37 weeks of gestation and the patient is not yet laboring, GBS cultures should be collected as above. Either of the following regimens may then be used:
        1. Give IV antibiotics until culture results are known, or
        2. Initiate antibiotic therapy only when culture result confirms presence of GBS.
  4. Care of the infant of a mother who has had GBS prophylaxis.
    1. Any infant with symptoms or signs of GBS and those infants born at less than 35 weeks gestation must have a full work-up (CBC, blood culture, CXR for pulmonary symptoms, LP if indicated). They should be treated until culture results are negative.
    2. For those >35 weeks without symptoms, approach is stratified based on duration of labor after the administration of antibiotics.
      1. If duration of labor after antibiotics is <4 hours, infant should have CBC, blood culture, and 48 hours of observation.
      2. If duration of labor after antibiotics is >4 hours, observation for 48 hours is indicated.

Obstetrics: Nausea and Vomiting of Pregnancy

  1. Cause. Unknown. Probably not related to serum HCG levels, but other hormones have been implicated (estradiol, thyroxine).
  2. Hyperemesis gravidarum. The incidence of hyperemesis gravidarum (severe nausea and vomiting causing ketosis and dehydration requiring hospitalization) is increased in multiple gestation and molar pregnancies, and so ultrasound is advisable in cases of hyperemesis gravidarum. Exclude organic causes: disorders of GI tract, gallbladder, pancreas, hepatitis, and urinary infection. Recent data suggests an association with Helicobacter pylori but this is not yet proven and treatment for H. pylori is not yet standard of care. Elevation of serum transaminase and mild jaundice can be observed, which will return to normal after adequate hydration and nutrition. Hyperemesis gravidarum has a 26% recurrence rate in subsequent pregnancies.
  3. Outpatient Management
    1. Reassurance that condition improves with time, usually by end of first trimester.
    2. Avoid medications whenever possible.
    3. Advise patient to arise slowly and to keep soda crackers at the bedside and eat before rising.
    4. Omit iron supplementation until nausea resolves.
    5. Eat frequent small meals and protein snacks at night.
    6. Antiemetics.
      1. Doxylamine succinate (Unisom) 25 mg 1/2 to 1 tablet PO Q am and Q pm. This can be used in conjunction with pyridoxine.
      2. Diphenhydramine (Benadryl) 25 to 50 mg PO Q6-8h.
      3. Phosphorylated carbohydrate (Naus-A-Way, Emetrol, Nausetrol) 15 to 30 ml PO on arising and Q3h PRN for nausea.
      4. Meclizine 25 to 100 mg PO BID to QID.
      5. Pyridoxine (vitamin B6) 25 mg PO TID.
      6. Bendectin (10 mg of doxylamine succinate and 10 mg of pyridoxine) was removed from the market, although large studies have not shown evidence of teratogenicity.
  4. Inpatient Management. For those with severe symptoms, weight loss, dehydration, ketones in urine, or high urine specific gravity. Correct hypovolemia, ketosis, and electrolyte imbalances with IV fluids. Use oral fluids as tolerated.
    1. Monitor fluid intake and output.
    2. Antiemetics as above; also consider phenothiazines: prochlorperazine (Compazine), droperidol (Inapsine) and promethazine (Phenergan).
    3. Parenteral nutrition for prolonged vomiting.
    4. Psychotherapeutic measures, stimulus control, biofeedback, and imagery can also be helpful.

Obstetrics: Diabetes in Pregnancy: Gestational Diabetes Mellitus (GDM)

  1. Potential morbidity.
    1. Infants born to diabetic mothers have 5 times the normal risk of respiratory distress syndrome, an increased risk of congenital anomalies (especially with first trimester hyperglycemia), an increased risk of neonatal hypoglycemia, hypocalcemia and jaundice.
    2. The mother has an increased incidence of preeclampsia, infection, postpartum bleeding, and cesarean section (secondary to macrosomia). There is also an increased risk of maternal injury during vaginal delivery.
  2. Evaluation.
    1. Glucose challenge test (GCT).
      1. Timing. A GCT is performed as a routine screen for GDM in all pregnancies at 24 to 28 weeks of gestation. It should be performed earlier if symptoms are present or if there is a previous pregnancy with GDM.
      2. If there are risk factors for GDM (e.g., massive obesity, prior history GDM), consider screening at the first prenatal visit if there are risk factors. Repeat at 24 to 28 weeks if initial test is negative.
      3. Procedure. A blood glucose level is obtained 1 hour after a 50 g oral glucose load.
      4. Interpretation. A level of 140 mg/dl or greater is abnormal. However, since there are many false positives, a 3-hour fasting glucose tolerance test should be done if GCT is >130 mg/dl.
    2. Glucose tolerance test (GTT).
      1. Indication. Follow-up of an abnormal GCT result.
      2. Procedure. The patient must eat a diet containing at least 150 g of carbohydrate for 2 days. Draw a serum glucose level after an overnight fast. The patient then ingests 100 g of glucose solution. Serum glucose levels are then obtained at 1, 2, and 3 hours.
      3. Interpretation (Table 14-1). If two or more of these readings are abnormal, the patient needs treatment for GDM.
    3. Management of gestational diabetes.
      1. Dietary adjustment is the mainstay of therapy.
        1. Caloric intake should be 30 to 35 kcal/kg/day. Intake should be reduced to 24 kcal/kg/day if the patient is obese.
        2. Avoid cakes, candy, and other fast-acting carbohydrates.
        3. Dietary composition should be 50% to 60% carbohydrate, 20% to 25% protein, and 20% fat, with high fiber content.
        4. Exercise has shown added benefit along with dietary therapy.
      2. Obstetric surveillance.
        1. Early ultrasound for accurate gestational dating.
        2. Follow every 2 weeks until 36 weeks and then weekly.
        3. Accucheck QID before meals and at bedtime.
        4. Check fasting blood glucose and review home monitoring at each visit. If fasting glucose levels are >105 mg/dl (or postprandial values are 120 to 130), the patient may be hospitalized to ensure adherence to diet. If fasting glucose remains >110 mg/dl, insulin therapy is indicated. Glyburide has been used safely after the first trimester in patients failing insulin. However, this is not the standard of care.
        5. Check for ketonuria daily to make sure there has been adequate caloric consumption.
        6. Obtain an ultrasound if macrosomia is suspected. If the estimated fetal weight is >4000 g, a cesarean section should be considered at term.
        7. Antepartum NST is often initiated on a weekly basis at 34 to 35 weeks of gestation but may be started earlier. If euglycemia can be documented, consider delaying monitoring until 38 weeks. Delivery is recommended if any sign of fetal compromise is noted.
        8. Amniocentesis is helpful in documenting fetal lung maturity before cesarean section, since infants of diabetic mothers have delayed lung maturity.
      3. Postpartum.
        1. Infants should be watched for evidence of hypoglycemia due to high levels of circulating insulin.
        2. Gestational diabetics should have a 75 g oral GTT checked 6 weeks postpartum to rule out persistent carbohydrate intolerance. Counsel the patient that she has an approximate 35% risk of developing diabetes at some point in her life.

Obstetrics: Trauma and Pregnancy

  1. Differential. Retrospective study in tertiary referral center showed etiology of trauma in pregnancy: 54% motor vehicle accidents (MVA), 22% domestic violence, 21% falls, 1.3% burn, puncture, or assault. Up to 50% of falls cause some degree of abruption.
    1. Physical or Sexual Abuse. Abused women tend to present late for prenatal care. Generally abuse continues and may worsen during pregnancy. Abused patients have an increased risk for: preterm labor, chorioamnionitis and low birth weight infants. Asking direct questions, such as "has anyone hit, slapped or kicked you, etc," is the best way of screening for abuse.
  2. Management.
    1. Treatment of pregnant woman. ABC’s of evaluating and stabilizing mom take priority. Deflect uterus away from great vessels by placing wedge under right hip or tilt table laterally. Once the mother has been stabilized, consider fetus. If mom is Rh negative, give RhoGam.
    2. Evaluation of fetus. Trauma increases the risk of placental abruption and fetomaternal hemorrhage. Fetal monitoring and contraction monitoring is the best way to assess for significant abruptio. Ultrasound is less sensitive and MRI is not generally practical (although very sensitive). Monitoring for 2-6 hours is sufficient if no ominous signs (vaginal bleed, contractions, and uterine tenderness) are noted. Make sure that the fetus has a good heart rate and good beat to beat variability as well. If contractions are not detected or occur less than every 10 minutes and monitoring of the fetus is normal, an abruption unlikely. Twenty percent of women with contractions greater than every 10 minutes have associated abruption.
    3. Return. Women should be instructed to return for abdominal cramps, increasing pain or vaginal bleeding.

Obstetrics: Hypertension in Pregnancy, Preeclampsia, and Eclampsia

  1. Pregnancy-Induced Hypertension (PIH).
    1. Definition. PIH is present when diastolic BP >90 mm Hg or a systolic BP >140 or a systolic BP rises at least 30 mm Hg over baseline value or diastolic BP rises at least 15 mm Hg over baseline value.
    2. Risk factors for PIH. First pregnancy, multiple gestation, polyhydramnios, hydatidiform mole, malnutrition, positive family history of PIH, underlying vascular disease. Molar pregnancy should be expected if PIH occurs early in gestation.
    3. Treatment. See section on preeclampsia and chronic hypertension.
  2. Preeclampsia and Eclampsia.
    1. Preeclampsia. Defined as the presence of hypertension or PIH accompanied by proteinuria, edema, or both after 20 weeks gestation. Preeclampsia is divided into mild and severe forms.
    2. Criteria for mild preeclampsia.
      1. Hypertension as defined above but not meeting the criteria for severe preeclampsia (below).
      2. Proteinuria >300 mg/24 hours.
      3. Mild edema, signaled by weight gain >2 lb/week or >6 lb/month.
      4. Urine output >500 ml/24 hours.
    3. Criteria for severe preeclampsia.
      1. BP of greater than 160/110 on 2 occasions at least 6 hours apart with patient on bed rest or a systolic BP rise of greater than 60 mm Hg over baseline value or a diastolic BP rise of greater than 30 mm Hg over baseline value.
      2. The presence of an elevated blood pressure and any of the systemic symptoms noted below categorizes the patient as having severe preeclampsia regardless of the blood pressure.
        1. Proteinuria >5 g/24 hours or 31 or 41 on urine dipstick.
        2. Massive edema.
        3. Oliguria <400 ml/24 hours.
        4. Systemic symptoms including pulmonary edema, headaches, visual changes, right upper quadrant pain, elevated liver enzymes, or thrombocytopenia.
        5. Presence of IUGR.
    4. Eclampsia: Occurrence of a seizure that is not attributable to other causes in a preeclamptic patient.
  3. Evaluation of PIH and Preeclampsia.
    1. History. Document risk factors and any symptoms outlined above.
    2. Physical. Look for evidence of edema (particularly of the hands and face), BP changes, retinal changes, hyperreflexia, clonus, and RUQ tenderness.
    3. Initial laboratory studies.
      1. Blood: CBC (elevated or normal Hb/HCT, low platelet count), electrolytes, BUN and creatinine (creatinine >1.0 ng/ml, BUN >10 mg/dl, uric acid (>5.5 ng/dl), liver function tests (elevated AST, ALT, LDH), and coagulation studies (elevated PT, PTT, and elevated fibrinogen degradation products). Patients may also have hypo-albuminemia, as well as schistocytes or helmet cells on peripheral smear from microangiopathic hemolysis. If patient is in labor, send a blood type and screen.
      2. Uric acid (>5.5 ng/dl) may be elevated before there are other signs or symptoms of preeclampsia.
      3. 24 hour urine for creatinine clearance, total protein.
  4. Complications of Preeclampsia. Eclamptic seizures; Hemolysis; Elevated Liver function tests, Low Platelet count (HELLP) syndrome, hepatic rupture, DIC, pulmonary edema, acute renal failure, placental abruption, intrauterine fetal demise (IUFD), cerebral hemorrhage, cortical blindness, retinal detachment.
  5. Management of PIH/Preeclampsia.
    1. Outpatient management. For pregnancy-induced hypertension without significant proteinuria, home bed rest is recommended. Home blood pressure monitoring, weight, and urine protein checks are helpful. Antepartum surveillance (NST) should begin early. Ultrasound exams should be performed periodically to ensure adequate amniotic fluid and to monitor for intrauterine growth retardation (IUGR).
    2. Hospital management.
      1. Indications. No improvement of mild pregnancy-induced-hypertension with home bedrest or pre-eclampsia with 2+ proteinuria, evidence of organ system involvement.
      2. Orders. Bed rest with bathroom privileges is allowed. The goal of IV fluids in severe cases is to replace urine output and insensible losses.
      3. Laboratory evaluation and weights. Performed daily to every other day. Antepartum surveillance including daily fetal movement count, daily NSTs, and weekly amniotic fluid determinations by ultrasound is essential. Monitor symptoms such as headache, visual disturbances, and epigastric pain.
      4. Delivery is treatment of choice. Delivery should be accomplished when the fetus is mature but may be required early if maternal health is in danger or if there is evidence of fetal distress. Delivery is indicated when the patient meets criteria for severe preeclampsia. Betamethasone 12.5 mg IM should be given twice 24 hours apart to stimulate fetal lung maturation and can be repeated weekly if pregnancy is prolonged. Electronic FHR monitoring during labor is indicated.
      5. Antihypertensive therapy. Indicated only if BP persistently >160/110. Aim for a diastolic BP 90 to 100 mm Hg. Avoid overcorrection because normal blood pressures can result in placental hypoperfusion.
        1. Diuretics and ACE inhibitor are contraindicated during pregnancy.
        2. Acute management: Hydralazine IV 5 mg bolus and infusion
        3. Long-term medications (if the fetus is immature) include methyldopa (Aldomet), atenolol, and labetalol.
      6. Anticonvulsant therapy. Seizure prophylaxis is indicated in all preeclamptic patients during labor and delivery and for a minimum of 24 hours postpartum. Seizures may occur in the absence of hyperreflexia, and increased DTRs may be present in the normal population; therefore, hyperreflexia is not a useful predictor of who will have a seizure.
        1. Drug of choice for seizure prophylaxis: Magnesium sulfate
        2. The loading dose is 4 to 6 g of magnesium sulfate IV over 20 minutes and continued at 2 g/hour.
        3. To treat active seizures. Magnesium sulfate 1 g/min IV until seizure controlled up to 4 to 6 g maximum. If this fails, see Chapter 2 for management of status epilepticus.
        4. Continue magnesium sulfate therapy at least 24 hours postpartum. In 25% of the patients postpartum eclampsia can occur. Monitor urine output postpartum and stop therapy if urine output is >200 ml/hour for 4 consecutive hours. Watch for postpartum hemorrhage because magnesium sulfate can relax the uterus.
      7. Managing magnesium therapy.
        1. Monitor urine output (100 ml in 4 hours), deep tendon reflexes and serum levels. Therapeutic level is 4 mEq/L but since it takes 12 to 18 hours to equilibrate, serum levels of magnesium sulfate are of dubious value.
        2. Magnesium toxicity. Loss of reflexes and drowsiness will herald magnesium toxicity. At levels of 10 to 12 mEq/L and above, muscle weakness, respiratory paralysis, and cardiac depression can occur.
        3. To treat magnesium toxicity. 10 ml of 10% calcium gluconate (or calcium chloride) may be administered IV push in the event of magnesium toxicity, or the magnesium infusion can be turned off for 1 to 2 hours.
  6. Prevention of Preeclampsia. Aspirin 81 mg a day and calcium supplementation have been used to prevent preeclampsia. However, they have not been shown to be of any benefit in controlled trials and do not change fetal or maternal outcomes.
  7. Chronic Hypertension Superimposed on Pregnancy.
    1. Risks.
      1. Maternal. The risk to the mother is the same as in the non-pregnant state. However, in the presence of superimposed preeclampsia (20%), there is increased maternal mortality, frequently from intracranial hemorrhage.
      2. Fetal. There is an increased incidence of perinatal death, IUGR, and fetal distress.
    2. Management.
      1. Treatment of chronic hypertension can decrease maternal and, to some extent, fetal morbidity. Appropriate medications include methyldopa, hydralazine, and beta-blockers.
      2. During pregnancy, it is not appropriate to use:
        1. Sympathetic ganglion blockers (orthostatic hypotension)
        2. Diuretics (aggravation of volume depletion)
        3. ACE inhibitors (associated with fetal defects and neonatal renal failure)
      3. Laboratory evaluation is performed early in pregnancy.
      4. Obstetric visits are scheduled every other week at 24 weeks and weekly after 30 weeks.
      5. Early ultrasound is obtained for dating, and repeated periodically to look for evidence of IUGR.
      6. Antenatal surveillance (NSTs) should begin at 34 weeks.
      7. Timing the delivery. The pregnancy should not be allowed to go beyond 40 weeks. Delivery may be required earlier if there is evidence of IUGR or fetal distress or if hypertension cannot be controlled by bed rest and medication.
      8. Intrapartum monitoring is required during labor.
      9. If there is evidence of IUGR, cesarean section is preferable to a prolonged induction.
      10. Complicated cases or women with superimposed preeclampsia should be handled at an appropriate referral center.

Obstetrics: Early Antepartum Hemorrhage

  1. Definition. Vaginal bleeding at <20 weeks of gestation.
  2. Differential Diagnosis of early vaginal bleeding.
    1. Spontaneous Abortion
      1. Incidence 15% to 25% of clinically recognized pregnancies end in a spontaneous abortion.
      2. Causes. Fetal abnormalities incompatible with life (chromosomal and other), defective implantation, maternal infection, uterine and cervical anomalies.
      3. Evaluation
        1. History. Suggestive of pregnancy (missed period or periods, nausea, vomiting, breast tenderness) followed by cramping and spotting or bleeding often with passage of tissue. All patients should be evaluated to rule out an ectopic pregnancy. Remember patients must be seen within 48 hours for RhoGAM if indicated (i.e., mother is Rh negative)
        2. Exam. Including stability of vital signs, orthostatic vital signs, pelvic exam looking for open or closed cervical os, tissue, other causes of vaginal bleeding (such as cervical eversion, polyp, infection, vaginal lesion, ectopic fetus). Size uterus. Check for fetal heart tones with Doppler scanning if 10 to 12 weeks.
        3. Lab tests
          1. Urine pregnancy test is positive in 75% of cases so a negative pregnancy test does not rule out spontaneous abortion.
          2. CBC, blood type, and antibody screen in all patients for Rh status. RhoGAM indicated for all Rh-negative, antibody- negative women.
          3. Uterine ultrasound or pathologic exam of tissue if indicated.
          4. Serial quantitative HCG should increase by at least 60% in 48 hours. If it does not rise or drops, it is likely that the pregnancy is nonviable.
    2. Threatened abortion. Vaginal bleeding ± cramps but with a cervix that is long and closed with a uterus appropriate for gestational age. Roughly 50% progress to inevitable abortion.
    3. Inevitable abortion. Persistent cramps and moderate bleeding and a cervical os is open. Do not confuse with an incompetent cervix, which is a painless cervical dilatation not associated with cramping and is potentially treatable.
    4. Incomplete abortion. The same symptoms as an inevitable abortion but with some retained products of conception in the uterus or cervical canal. There is ongoing cramping and excessive vaginal bleeding. Speculum examination reveals a dilated internal os and tissue present within the endocervical canal or vagina. Bleeding may be heavy and clots may be mistaken for products of conception.
    5. Complete abortion. The entire conceptus is expelled and cramping and bleeding abate or resolve completely. On examination, the uterus is firm, and smaller than one would expect for gestational age.
    6. Missed abortion. Products of conception retained 3 or more weeks after fetal death. Signs and symptoms of pregnancy abate; pregnancy test becomes negative. Brownish vaginal discharge (rarely frank bleeding) occurs. Cramping is rare. The uterus is soft and irregular. Ultrasound exam rules out live pregnancy.
    7. Septic abortion. Any of the above scenarios and a temperature of greater than 38° C without other source of fever. Septic abortion is associated with (but does not require) IUD use or instrumentation during abortion. Abdominal and uterine tenderness are present as well as purulent discharge and possibly shock.
    8. Ectopic Pregnancy. See Chapter 13.
    9. Molar pregnancy (hydatidiform mole). Placenta undergoes trophoblastic proliferation and typically resembles a cluster of grapes. Occurs more often in women less than 20 or greater than 40 years of age and almost always causes some degree of vaginal bleeding. Hydatidiform moles are associated with hyperemesis gravidarum and the onset of preeclampsia before the third trimester. The uterus is larger than expected for gestational age in 50% of the cases. Ovarian enlargement may occur secondary to thecal lutein cysts. Ultrasound findings typically show a "snowstorm" pattern. Nearly 20% of hydatidiform moles progress to gestational trophoblastic tumor. Treatment: Immediate evacuation of mole, subsequent follow-up for detection of persistent trophoblastic proliferation or malignant change
  3. Treatment.
    1. Assure adequate circulating volume. Treat with IV normal saline or lactated Ringer’s. Consider transfusion if Hb <8 g or patient is unstable.
    2. Threatened abortion. Bed rest if possible; use acetaminophen for discomfort, nothing in the vagina (no tampons, douches, intercourse), consider ultrasound for gestational sac, cardiac activity, or to rule out ectopic pregnancy. Positive cardiac activity predictive of continued pregnancy >90%. Consider monitoring quantitative beta-hCG with a rise of less than 66% in 48 hours predictive of abortion or ectopic.
    3. Incomplete or inevitable abortion. Hospitalize if hypovolemic, anemic, or advanced gestation >12 weeks. Tissue visible in os should be gently removed with ring forceps to allow contraction of uterus; but minimize manipulation to decrease risk of infection. Patients with incomplete abortion (tissue passed with continued bleeding) often require suction curettage or D&C. Consider oxytocin drip as an alternative (20 IU in 1000 ml of crystalloid solution at 50 to 100 ml/hour). If unsuccessful, proceed with D&C.
    4. Complete abortion. Discharge home if vital signs stable, Hb documented to be stable, and bleeding decreased. Consider methylergo-novine (Methergine) 0.2 mg PO TID for 3 days if diagnosis certain or after uterine evacuation.
    5. Missed abortion. Obtain CBC with differential, platelet count, PT and PTT, and DIC (see Chapter 5) panel if indicated. Outpatient management may be considered if retained for less than 4 weeks, if weekly fibrinogen levels are obtained, and if the patient is monitored closely for DIC. Hospitalize if there are signs of infection, DIC, or if the fetus has been retained longer than 4 weeks. Fibrinogen levels of less than 150 mg/dl call for immediate evacuation of the uterus.
    6. Septic abortion. Obtain CBC, UA, culture of discharge from uterus, blood cultures, chest radiograph for diagnosis of septic emboli and to rule out free air from perforation and abdominal radiograph to evaluate for uterine foreign body. Electrolytes and ABG. Organisms include both anaerobes and aerobes (Bacteroides, Streptococcus, Enterobacter, Chlamydia, Clostridium). Hospitalize, treat sepsis, D&C, IV antibiotics:
      1. Doxycycline plus cefoxitin or imipenem or ticarcillin, or
      2. Clindamycin plus third-generation cephalosporin or gentamicin
      3. Discharge to home with taking oral doxycycline or clindamycin.
    7. Long-Term Management Give RhoGAM to Rh-negative women. Provide emotional support. Traditional but not well-founded recommendation is to wait 3 months before attempting conception. Having a single spontaneous abortion does not increase the risk of aborting the next pregnancy. Evaluate couple for habitual abortion if the woman has had two or more successive spontaneous abortions. If the patient is a habitual aborter, obtain antiphospholipid antibody titers. Obtain fetal tissue for karyotyping if possible.

Obstetrics: Late Antepartum Hemorrhage

  1. Definition. Vaginal bleeding that occurs after 20 weeks of gestation.
  2. Differential Diagnosis.
    1. Placenta previa.
      1. Incidence. Occurs in 1 of 200 deliveries. The diagnosis of placenta previa is very common in the second trimester, but more than 95% of these do not have placenta previa at delivery.
      2. Classification. Placenta previa may be marginal, partial, or total depending on how much of the placenta is over the cervical os.
      3. Diagnosis. Vaginal bleeding is typically bright red and painless. The blood loss is not massive but tends to recur and become heavier as the pregnancy progresses. Diagnosis may be made by ultrasound. The advisability of a speculum exam is debatable. Digital examination is contraindicated other than in a double setup situation when delivery is desirable and can be rapidly accomplished by C-section. Maternal risk factors include increasing age, multiparity, and prior uterine scar. Associated with breech and transverse presentations.
    2. Placental abruption.
      1. Incidence. Placental abruption occurs in 10% of all deliveries. Severe abruption is rare.
      2. Classification.
        1. Mild. Slight vaginal bleeding (<100 ml), no FHR abnormalities are present; there is no evidence of shock or coagulopathy.
        2. Moderate. Moderate vaginal bleeding (100 to 500 ml) and uterine hypersensitivity with or without elevated tone. Mild shock and fetal distress may be present.
        3. Severe. Extensive vaginal bleeding (>500 ml), tetanic uterus, and moderate to profound maternal shock are present. Fetal demise and maternal coagulopathy are characteristic.
      3. Diagnosis. The diagnosis of placental abruption is clinical. Although vaginal bleeding is present in 80% of cases, bleeding may be concealed in the remainder (that is, retroplacental bleeding). Thus, the maternal hemodynamic situation may not be explained by observed blood loss. Pain and increased uterine tone are typically present. Risk factors include prior history of abruption, maternal hypertension, cigarette or cocaine use, increasing maternal age or multipar-ity. Abruption may be associated with premature rupture of membranes, blunt abdominal trauma, and twin gestation after delivery of first infant.
    3. Uterine rupture. Very rare. May mimic severe abruption. An abdominal film may show free intraperitoneal air or an abnormal fetal position. Accompanied by persistent fetal bradycardia. Emergent C-section and hysterectomy are required.
    4. Other. Vasa previa (velamentous insertion of the cord). Delivery should be by scheduled C-section. If pregnancy is allowed to progress to term, spontaneous rupture of membrane or amniotomy should be averted because it could lead to fatal bleeding for fetus and possibly mother. Cervical dilation with loss of mucus plug may be confused with other causes of vaginal bleeding or cervical or vaginal lesions (polyps, condylomas).
  3. Laboratory Evaluation should include a CBC, type and cross, coagulation studies, urinalysis, and ultrasound.
  4. Management of Placenta Previa and Placental Abruption.
    1. Placenta previa.
      1. If pregnancy 37 weeks or greater, or if fetal maturity has been documented, a cesarean section is indicated unless only a minimal degree of placenta previa is present.
      2. If bleeding is sufficient to jeopardize the mother or fetus despite transfusion, cesarean section may be indicated regardless of gestation.
      3. In the preterm gestation, expectant management is indicated in patients with no observed bleeding, reactive nonstress test and stable hematocrit, who are compliant with instructions. Most patients require inpatient observation. Physical activity is restricted. Nothing is allowed in the vagina, including examining fingers. The hematocrit is maintained at 30% or greater. Preterm labor can be managed with magnesium sulfate. Use of beta-adrenergic agents can cause tachycardia and mask the signs of bleeding. Once 36 to 37 weeks of gestation is reached with fetal maturity demonstrated by amniocentesis, the patient is readied for elective double-setup examination.
      4. Check for fetal bleeding: To 5 ml of tap water add 6 drops of 10% KOH in two test tubes. Add 3 drops of maternal blood to one tube and 3 drops of vaginal blood to the other. The maternal blood will turn green yellowish brown after 2 minutes. If fetal red blood cells are present, the solution will turn pink. Immediate delivery is indicated.
      5. Remember that placenta accreta may complicate placenta previa in women with history of previous C-section. Hemorrhage can necessitate hysterectomy.
    2. Placental abruption.
      1. Occasionally a small separation occurs without further problem. These patients have no uterine symptoms. Observation is required with fetal heart rate monitoring, serial labs and ultrasound, but if no fetal distress occurs within the next 48 hours, the patient may be sent home.
      2. If placental abruption is mild and the fetus is immature, expectant management may be indicated, with fetal heart rate monitoring and serial laboratory and ultrasound examination.
      3. In all other cases, delivery is indicated. A vaginal delivery is preferred when fetal distress is not present or when the fetus is no longer viable. A C-section is indicated if fetal distress is present. A C-section is also performed when there is a threat to the mother’s life or a failed trial of labor.
      4. Shock must be treated with adequate replacement of fluids and packed red blood cells; NS or LR should be used. Urine output must be maintained at 25 to 30 ml/hour. A central venous pressure line or Swan-Ganz catheter will assist in monitoring hemodynamic status. See section on shock in Chapter 2.
      5. Coagulopathy should be treated with fresh frozen plasma. One unit of FFP increases the fibrinogen concentration by 25 mg/dl. Platelet transfusion is required if the count is less than 50,000. Heparin is not used in DIC secondary to placental abruption. See section on DIC in Chapter 6.

Obstetrics: Intrauterine Growth Retardation (IUGR)

  1. Definition. IUGR is defined as a fetus that weighs less than the tenth percentile for its gestational age.
    1. Symmetric IUGR (intrinsic): normal head circumference-to-abdominal circumference ratio, caused by genetic disease or fetal infection and has a poor prognosis.
    2. Asymmetric IUGR (extrinsic): increased head circumference/abdominal circumference ratio, caused by placental insufficiency; good prognosis with appropriate treatment.
  2. Risk Factors.
    1. Chronic maternal disease including chronic maternal hypertension, PIH, diabetes, cyanotic heart disease, collagen vascular disease, severe maternal anemia, renal disease, multifetal pregnancy, etc.
    2. Fetal genetic disorders or fetal malformations.
    3. Intrauterine infections. Rubella, herpes, toxoplasmosis, syphilis, CMV.
    4. Previous history of small-for-gestational-age baby, smoking, drug, or alcohol abuse.
    5. Abnormalities of the placenta or placental blood flow.
  3. Diagnosis. One should be suspicious when the fundal height does not exhibit the predicted 1 cm/week growth between 20 and 36 weeks of gestation. A lag in fundal height by 4 cm mandates ultrasonographic evaluation; otherwise, consider ultrasound on a clinical basis. Serial ultrasonic scanning may confirm the diagnosis.
  4. Management. The development of IUGR makes the pregnancy high risk. Stillbirth, oligohydramnios, and intrapartum fetal acidosis are common antepartum complications. Close antepartum surveillance is required, and the decision about when to deliver the infant is complex. Neonatal complications include persistent fetal circulation, meconium aspiration syndrome, hypoxic ischemic encephalopathy, hypoglycemia, hypocalcemia, hyperviscosity, and defective temperature regulation. A perinatologist should manage these pregnancies.

Obstetrics: Labor

Introduction. This section is organized sequentially as events happen during labor and delivery. It starts with the management of preterm labor and postdate pregnancies. It then discusses the stages and management of labor, as well as the induction of labor. Finally, it discusses the delivery itself.

Preterm Labor

  1. Definition. Onset of regular contractions between 20 and 37 weeks of gestation occurring at least every 10 minutes and lasting 30 seconds with cervical change. Discrimination from "false labor" is difficult. Postponement of treatment until cervical change occurs may lower the chances of success.
  2. Causes. Frequently unknown. Several factors have been associated with preterm labor.
    1. Maternal factors. Infections (systemic, vaginal, urinary tract, amnionitis), uterine anomalies, fibroids, retained IUD, cervical incompetence, overdistended uterus (polyhydramnios, multiple gestation), rupture of membranes.
    2. Fetal factors. Congenital anomalies, intrauterine death.
  3. Management.
    1. Initial examination.
      1. Estimate fetal weight and age by ultrasound if necessary.
      2. Document FHR and uterine activity with external monitoring.
      3. Pelvic examination. Attempt to limit to one examiner and use sterile technique. Rule out ruptured membranes by looking for vaginal pooling of amniotic fluid and by nitrazine paper testing (turns blue if amniotic fluid present) and evaluate sample of fluid for ferning via microscope. Obtain cervical cultures for group B streptococci and do rapid group B streptococci antigen testing if available. If membranes are ruptured, one can used pooled amniotic fluid to determine fetal maturity by looking at the L/S ratio and PG levels; otherwise amniocentesis may be necessary.
      4. Obtain cath UA and culture.
      5. Consider fibronectin test-swab of posterior vagina.
    2. Tocolysis.
      1. Contraindications. Evidence of fetal distress, fetal anomalies, abruptio placentae, placenta previa with heavy bleeding, severe maternal disease.
      2. Risks of treatment. If membranes are ruptured, there is increased risk of cord prolapse and amnionitis. Fetal mortality is increased if labor is suppressed when there is IUGR. Mother may experience tachycardia, nervousness, or pulmonary edema secondary to medication.
      3. Tocolysis most likely will be ineffective if labor is well established or if the cervix is dilated to 4 cm or more. Preparation should be made to deliver in the optimal setting. Up to now there have been no large-scale controlled clinical trials demonstrating that tocolytics delay delivery.
      4. Beta-adrenergic receptor agonists may inhibit uterine contractility but only prolong gestation for about 48 hours. To a large extent, the goal of tocolysis is to arrest labor long enough for exogenous steroids to stimulate fetal surfactant production so as to prevent the pulmonary complications of preterm birth.
        1. Protocol.
          1. Bed rest in left lateral decubitus position. Effective alone in 50% of patients.
          2. Sedation (100 mg of secobarbital or 50 mg of hydroxyzine).
          3. Hydration, but avoid large boluses (should not exceed 500 ml).
          4. Antibiotics controversial. Do not use for >2 days, to limit incidence of resistance.
          5. FHR and uterine activity monitoring.
          6. Steroids accelerate fetal lung maturation (betamethasone or dexamethasone 12.5 mg IM Q24h for 48 hours).
        2. Drug therapy (tocolytics)
          1. Terbutaline. Infusion should be titrated on an individual basis so as to maximize inhibition of uterine activity and minimize maternal side effects. Alternative to infusion: 0.25 mg SQ Q20-60 min until contractions have subsided. Continue 2.5 mg PO every 2 to 4 hours. Doses up to 5.0 mg can be used.
          2. Magnesium sulfate. MgSO4 also decreases uterine contractility but is not useful long-term. It can be an adjunct to terbutaline.
          3. Other. Prostaglandin synthetase inhibitors (such as indo-methacin), calcium-channel blockers, aminophylline, and progesterone are under investigation.

Premature Rupture of Membranes (PROM)

  1. Definitions.
    1. "Premature" rupture of membranes occurs if there is a delay of greater than 1 hour until onset of labor.
    2. "Preterm premature" rupture of membranes occurs before 37 weeks of gestation.
  2. Diagnosis.
    1. History of fluid gush per vagina. Urine can sometimes be confused with the rupture of membranes.
    2. Sterile speculum exam.
      1. Exam will show pooling of fluid in vaginal vault.
      2. pH determination. Amniotic fluid typically turns nitrazine paper blue. Contamination with vaginal-cervical mucus, blood, or urine may lead to false positives.
      3. Fern test. Allow a sample of fluid to air dry on a glass or slide. Examination of amniotic fluid under the microscope reveals a classical "fern" pattern.
      4. Cervical digital examination increases risk of chorioamnionitis! Evaluate cervix visually with sterile speculum. Avoid digital exams if possible unless patient is in labor and delivery is inevitable. Check for cord prolapse.
  3. Management.
    1. PROM at term. Most sources recommend induction and delivery within a range of 24 to 36 hours after admission.
    2. Preterm PROM. Fetal maturity must be considered. Manage expectantly until the fetus is mature unless chorioamnionitis (see below), fetal distress develops, or labor cannot be inhibited with tocolysis (see above). Positive cervical cultures should be treated but do not necessitate induction without other signs of chorioamnionitis or fetal distress. Follow maternal and fetal vital signs, including temperature every 8 hours and WBC counts as indicated. Antibiotics have been shown to prolong pregnancy. Randomized double blind placebo controlled study showed decreased infant morbidity with antibiotic therapy (ampicillin and erythromycin).
    3. Deliver if amnionitis. Signs include maternal or fetal tachycardia, maternal fever, uterine tenderness, foul cervical discharge, uterine contractions, leukocytosis, and the presence of leukocytes or bacteria in amniotic fluid.

Postdate Pregnancy

  1. Definitions.
    1. Prolonged pregnancy. Longer than 40 weeks of gestation.
    2. Postdate pregnancy. Longer than 42 weeks.
    3. Postmature pregnancy. Longer than 42 weeks with evidence of placental dysfunction.
  2. Etiology.
    1. Most common. Error in estimating EDC.
    2. Risk factors. History of prior prolonged gestation (50% risk), older age, anencephaly, or fetal endocrinopathy.
  3. Potential Morbidity.
    1. Maternal.
      1. Birth trauma secondary to macrosomic infant because of shoulder dystocia.
      2. Operative delivery, secondary infection and hemorrhage are more common with postdate pregnancies.
    2. Neonatal. Meconium aspiration syndrome, polycythemia, hyperbilirubinemia, hypoglycemia, and anoxic organ damage.
  4. Management: Delivery Should Be Accomplished by 42 Weeks.
    1. Antepartum fetal surveillance with NST and amniotic fluid index assessment should be done at 40 and 41 weeks and twice weekly thereafter.
    2. Indications for immediate delivery. Cervix is ripe, decreased amniotic fluid, large fetal size (abdominal circumference), nonreactive NST, and presence of meconium in fluid. Pregnancies complicated by hypertension and diabetes should be induced at or near term.
    3. Induction of labor can be preceded by cervical ripening using PGE2 gel or insert. PGE2 gel (1 mg placed intracervically) has been shown to decrease the amount of oxytocin needed to establish labor and rate of cesarean section in patients induced for medical indications before 41 weeks of gestation. Decreased amniotic fluid leading to variable decelerations and meconium staining may be managed with amnioinfusion (see below). Nasopharyngeal aspiration at the perineum and endotracheal aspiration should be performed once the baby is born to prevent meconium aspiration. Anticipate shoulder dystocia.

Vaginal Birth After Cesarean Section (VBAC)

  1. Definition. Attempted vaginal delivery in a woman who has undergone previous cesarean section.
  2. Decision to Attempt VBAC.
    1. Advantages include overall reduced morbidity, mortality and cost compared with elective C-section. Additionally, many women prefer a vaginal delivery.
    2. Disadvantages include the requirement for closer intrapartum monitoring and a higher risk of infection compared to an elective C-section if a C-section is required.
    3. Contraindications include a history of previous classical, T-shaped, or unknown uterine incision, multiple gestation, an estimated birth weight >4000 g, a non-vertex presentation or inadequate facilities or personnel for emergency C-section.
    4. Probability of success. Depends primarily on the indication of the previous C-section. If the primary C-section was for breech position, abruption, placenta previa, cord accident, antepartum hemorrhage, hypertensive disorder, or fetal distress, there is a 74% to 94% rate of success. If the primary C-section was for cephalopelvic disproportion (CPD) or failed induction, there is a 35% to 77% rate of success.
  3. Risks. In addition to the usual risk of delivery, other risks include:
    1. Uterine rupture. Very rare. Incidence increased if prior C-section was classical.
    2. Cesarean section. Increased risk of C-section morbidity relative to elective C-section.
  4. Management.
    1. Preparation.
      1. Type and screen for 2 units of packed cells; intravenous line should be inserted.
      2. The anesthesiologist, surgeon, and physician caring for the newborn infant must be notified in advance and be available.
    2. Labor.
      1. Electronic fetal monitoring is recommended.
      2. Oxytocin may be cautiously used to augment labor, and close monitoring of uterine contractions (using intrauterine pressure catheter) is necessary. Oxytocin must be titrated with great care in a VBAC.
      3. The same expectations of normal progression during labor should be applied to patients with a prior C-section.
      4. An experienced physician should be in attendance throughout labor and delivery.
      5. Postpartum. Manual exploration of the uterus after delivery of the placenta is indicated to assess scar integrity.

Evaluation of Labor (Box 14-1)

  1. Collect the information in Box 14-1 on admission to labor and delivery.
  2. Pelvic Exam.
    1. Inspection.
      1. Look for herpetic lesions, condylomas, and lacerations.
      2. Speculum examination may reveal pooling of vaginal fluid, consistent with rupture of membranes. A nitrazine paper test or swab of vaginal fluid on a glass slide may be necessary to prove the presence of amniotic fluid in the vagina. The basic pH of this fluid will turn the nitrazine paper blue. Care must be taken to avoid the cervical mucus, which is also basic and may give a false-positive test. If an air-dried sample of fluid reveals a fern-like pattern, the presence of amniotic fluid is confirmed.
    2. Palpation of the cervix.
      1. Dilatation of the cervical os. Dilatation may range from 0 to 10 cm.
      2. Effacement. The degree of thinning of the cervix. The cervix may range from 3 cm long (thick or with no effacement) to paper thin (100% effaced). In nulliparas effacement often precedes dilatation. Simultaneous effacement and dilatation is seen in multiparas.
      3. Palpation of the presenting part.
        1. Identification. Head, foot, buttock, other.
        2. Station. Station is described as the relationship of the fetal presenting part to the level of the ischial spines in the maternal pelvis. Station may range from -3 to +3. Zero station (engagement) occurs when the lower most presenting part is palpable at the level of the ischial spines. Always assess station by both abdominal method and pelvic method to avoid errors caused by caput.
        3. Position. Position is described as the orientation of the presenting part in regard to the maternal pelvis. Vertex presentation with the occiput positioned either to the right or left anteriorly is the most common.

Normal Labor and Labor Dysfunction

  1. Normal Phases of Labor.
    1. Latent phase. Slow rate of dilatation, <0.6 cm/hour.
    2. Active labor.
      1. Acceleration. Dilatation rate >0.6 cm/hour.
      2. Maximum slope of dilatation. Cervix >5 cm or rate >1.2 cm/hour for nullipara and >1.5 cm/hour for multipara.
      3. Deceleration. Cervix >9 cm, not completely effaced.
  2. Problems with the Progression of Labor.
    1. Prolonged Latent Phase. Defined as >20 hours in nullipara; >14 hours in multipara. Cause: unripe cervix, false labor, sedation, and uterine inertia. Management: observation, need for oxytocin stimulation. Avoid amniotomy. Good prognosis for vaginal delivery.
    2. Protracted Active Phase. Rate of dilatation: <1.2 cm/hour in nulli-para; <1.5 cm/hour in multipara. Cause: fetal malpositions (occi- put posterior), CPD, hypotonic uterine contractions, and anesthesia. Management: oxytocin stimulation. 70% require C-section.
    3. Secondary Arrest of Cervical Dilatation. Cessation of dilatation for >2 hours. High incidence of CPD: frequently require a cesarean section.
    4. Failure of Descent. Arrest of descent during second stage. High incidence of CPD: frequently required cesarean section.
    5. Protracted Descent. Nullipara <1 cm/hour; multipara <2 cm/hour. Causes include CPD, full bladder, and macrosomia. Inadequate pushing because of anesthesia can also cause this disorder.
    6. Precipitous Labor >5 cm/hour dilatation in nullipara; >10 cm/hour in multipara. Complications: trauma to birth canal, fetal distress, and postpartum hemorrhage.

Intrapartum Monitoring and Management

  1. Fetal Heart Rate. Electronic fetal heart rate monitoring may be performed by means of external Doppler, or direct scalp lead when membranes are ruptured.
    1. Indications. Meconium staining, use of oxytocin; delivery of an anticipated premature, postmature, Rh-sensitized, or growth-retarded infant; medical complications associated with uteroplacental insufficiency (hypertension, diabetes, severe anemia, heart disease, renal disease), presence of abnormal FHR by Doppler scanning, VBAC, other intrapartum obstetrical complications (failure to progress, excessive vaginal bleeding).
    2. Fetal heart rate tracing interpretation.
      1. Baseline fetal heart rate.
        1. Normal 120 to 160 bpm.
        2. Tachycardia >160 bpm. Cause: fetal hypoxia, maternal fever, maternal hyperthyroidism, parasympatholytic or sympathomimetic drugs.
        3. Bradycardia <120 bpm. Cause: fetal asphyxia, anesthetics, fetal cardiac conduction defect. Usually benign if good variability is present.
      2. Variability.
        1. Short-term variability. Beat-to-beat variation is normally 5 to 10 bpm (reliably assessed with only a scalp lead).
        2. Long-term variability. Waviness of the FHR tracing, which normally has a frequency of 3 to 10 cycles/min and an amplitude of 10 to 25 bpm.
        3. Decreased variability. Variability may be decreased by fetal sleep cycles, CNS depression secondary to hypoxia or drugs, parasympatholytic agents, extreme prematurity, or congenital anomalies. Loss of variability is associated with a high incidence of fetal acidosis and low Apgar scores.
      3. Common periodic patterns.
        1. Accelerations. Reassuring if associated with fetal movement. May be compensatory before or after deceleration.
        2. Early decelerations. Occur coincidentally with uterine contractions and are associated with fetal head compression. These are vagally mediated and not ominous when they occur late in labor. These start early in the contraction phase, reach their lowest point at the peak of the contraction, and return to baseline levels as the contraction finishes. The FHR does not fall below 100 bpm.
        3. Late decelerations. Transient but repetitive deceleration of the FHR observed to occur late in the contraction phase. Reaches its lowest point after the acme of the contraction and returns to baseline rate once the contraction is over. Late decelerations result from fetal hypoxia, indicate uteroplacental insufficiency, and are always considered ominous.
        4. Variable decelerations. Characterized by variable duration, timing in relation to contraction and intensity. This is a reflex pattern, typically secondary to umbilical cord compression. May benefit from amnioinfusion. Poor prognostic signs are the following:
          1. Association with poor FHR baseline variability.
          2. Lack of pre-deceleration and post-deceleration accelerations.
          3. Slow return to baseline or failure to return to baseline.
          4. Biphasic shape (W = knot in cord).
        5. Prolonged decelerations. Isolated decelerations >120 seconds can be seen with maternal hypotension, maternal hypoxia, tetanic contractions, prolapsed umbilical cord, fetal scalp procedures (vagal), and paracervical or epidural anesthesia. A prolonged deceleration after severe variable deceleration may signal impending fetal demise.
      4. Management of abnormal FHR pattern or fetal distress.
        1. Turn patient onto left side to alleviate vena cava compression.
        2. Discontinue intravenous oxytocin.
        3. Apply 100% oxygen to mother by face-mask.
        4. Correct maternal hypotension or hypertension.
        5. Vaginal examination to rule out prolapsed cord.
        6. Consider fetal scalp blood sampling for pH determination (Table 14-2).
        7. With decreased variability, consider fetal scalp stimulation. The return of variability is reassuring. If tracing maintains poor variability, consider points a to f above.
        8. With prolonged bradycardia unresponsive to other maneuvers or late decelerations with worsening fetal acidosis (pH <7.20), consider delivery by C-section.
  2. Uterine Activity. May be determined by an indirect (external) pressure monitor, or by an intrauterine pressure transducer when more accurate estimations are required.
    1. Contractility. Effective contractions should have an amplitude of 50 to 75 mm Hg, duration of 45 to 90 seconds, and frequency of every 3 to 5 minutes.
    2. Resting tone. Spontaneous labor 5 to 10 mm Hg. Induced labor 15 to 20 mm Hg.
    3. Rhythmicity. Presence of coupling or tripling may represent hyperstimulation.
    4. Configuration. Typically bell shaped. May become rectangular during pushing. The area under the curve when an internal transducer is used may be calculated to determine the adequacy of uterine contractions.
  3. Fetal Stimulation. When the scalp is stimulated and there is an acceleration of 15 bpm lasting 15 seconds, it denotes fetal pH value of 7.22 or greater. Reverse is not true. Obtain baseline fetal scalp pH in meconium staining. Draw maternal venous blood simultaneously for comparison. In the case of maternal fever do not rely on fetal scalp pH because fetal compromise can occur with normal values.

Obstetrics: Amnioinfusion

  1. Definition. Amnioinfusion is a procedure in which a physiologic solution (such as normal saline) is infused into the uterine cavity to replace the amniotic fluid.
  2. Indications.
    1. Correcting variable decelerations because of cord compression.
    2. Reduce fetal distress caused by meconium staining of fluid (rule out concurrent signs of fetal stress).
    3. Correction of oligohydramnios.
  3. Technique.
    1. Catheter. Double-lumen catheter: expensive but helps monitor uterine contractions.
    2. Infusate. Normal saline, lactated Ringer’s (like amniotic fluid).
    3. Temperature. Room temperature fluid can cause fetal bradycardia if infused rapidly. Body temperature is more physiologic.
    4. Methods. Continuous infusion by gravity drainage or by infusion pump 10 to 15 ml/min or intermittent infusion by gravity drainage (1 L over 20 to 30 minutes, repeat Q6h). Small risk of uterine rupture if efflux of infusate blocked.
  4. Efficacy.
    1. Oligohydramnios. Lower rate of C-section for fetal distress and higher umbilical artery pHs at birth compared to those in patients not receiving amnioinfusion.
    2. Moderate to thick meconium. Decreased rate of operative delivery, increased average 1-minute Apgar scores, less meconium aspirated from below neonate’s vocal cords, and a lower incidence of meconium aspiration syndrome compared to that in patients not treated with amnioinfusion.

Induction of Labor

  1. Indications and Contraindications.
    1. Indications. Pregnancy-induced hypertension, premature rupture of membranes, chorioamnionitis, postdate pregnancy, IUGR, isoimmunization, other evidence of hostile intrauterine environment, diabetes mellitus, other selected maternal diseases, fetal demise.
    2. Contraindications. Placenta previa, cord presentation, floating presenting part, abnormal fetal lie, active genital herpes, invasive cervical carcinoma, pelvic structural deformities, prior classical uterine incision. Oxytocin stimulation would be relatively contraindicated in conditions that predispose to uterine rupture (high parity, advanced maternal age, fetopelvic disproportion, uterine overdistension, prior uterine scar).
  2. Induction Methods. Assess the inducibility of the cervix using Bishop score (Table 14-3). Determine route of induction.
    1. Amniotomy.
      1. Cervix should be dilated enough to allow reaching the membranes with the amniotomy hook. The fetus should be vertex (unless breech delivery is planned) with the presenting part well engaged and well applied to the cervix. The umbilical cord should not be palpable.
      2. Membranes are hooked, and a gentle tug should cause release of amnionic fluid. Assess fluid for presence of meconium.
      3. Monitor fetal heart tones before and after the procedure.
      4. Risks: cord prolapse, injury to fetal part (unlikely with amnio hook)
    2. Dinoprostone (PGE2)
      1. Indicated for cervical ripening if Bishop score <5 (see Table 14-3)
      2. Cervidil (10 mg dinoprostone). Administer as vaginal insert. 1 dose. Monitor for 120 minutes postinsertion.
      3. Prepidil (0.5 mg dinoprostone) Administered intracervically. Mon- itor for 60-120 minutes. May repeat after 6 hours. No more than 3 doses in 24 hours.
      4. Risks. Hyperstimulation and uterine rupture.
      5. Endpoint. Bishops score of 8 or higher, strong uterine contractions or change in maternal/fetal status.
    3. Misoprostol (PGE1)
      1. Currently not FDA-labelled for cervical ripening but meta-analysis compared use of intravaginal misoprostol with dinoprostone, oxytocin, and placebo. Lower rate of cesarean section, higher incidence vaginal delivery within 24 hours, and reduced need for oxytocin augmentation.
      2. Most common dose 25-50 mg inserted every 4-6 hours up to 6-8 doses. Continuous fetal monitoring recommended for 3 hours after dose.
      3. Risks. Hyperstimulation, possible increased meconium staining, uterine rupture
    4. Oxytocin administration.
      1. Can use to ripen cervix but often used after one of the above methods of cervical ripening.
      2. Close monitoring of the parturient and fetus is essential. Most hospitals have written protocols available.
      3. Place 10 units of oxytocin in 1000 ml of D5NS or D5LR. Begin with a low dose of oxytocin: 0.5 to 2 mU per minute. (Each milliliter of the above solution contains 10 mU.).
      4. Advancing dose. Various protocols exist regarding the rate for increasing the dose and the maximum dose. If little uterine response is observed, the dose can be increased by 1 to 2 mU/min every 30 minutes. Most patients respond to rates of 20 mU/min or less. The faster the increase, the more likely the risk of hyperstimulation. The rate of administration is held steady when a good labor pattern (contractions every 2 to 3 minutes lasting 60 to 90 seconds with an intrauterine pressure of 50 to 60 mm Hg and a resting tone of 10 to 15 mm Hg) is achieved. Ideally you want 150 to 250 Montevideo units. Montevideo units = Number of contractions/10 min x (Average peak of contraction - Average baseline of contraction).
      5. If at any point the fetal heart rate indicates distress, the patient should be placed on her left side, oxygen administered, and oxytocin discontinued. Reinstatement of oxytocin drip requires reassessment of the situation

Obstetrics: Obstetric Anesthesia and Analgesia

  1. Overview. Pain during first stage of labor is attributable to uterine contractions and cervical dilatation. During the second stage, pain occurs from distension and stretching of pelvic structures and the perineum. Pain is conducted along the paracervical or inferior hypogastric plexus.
  2. Systemic Narcotics. Meperidine 25 mg IV or nalbuphine (Nubain) 10 mg IV are given early during labor and usually avoided at or near delivery. Maternal complications: nausea, vomiting, decreased gastric motility, respiratory depression. Fetal complications: respiratory depression, CNS depression, and impaired temperature regulation. Naloxone (0.01 mg/kg) can be administered to depressed newborn as IV bolus for counteracting the effect of narcotics. Naloxone may also be given IM.
  3. Local Anesthesia.
    1. Pudendal block. Provides analgesia to vaginal introitus and perineum. Usually used in second stage of labor. Technique is beyond the scope of this manual.
    2. Paracervical block. Provides analgesia during active phase of labor. Blocks pain caused by uterine contractions. Technique is beyond the scope of this manual.
    3. Lumbar epidural anesthesia.
      1. Associated with prolonged labor and an increased risk of chorioamnionitis.
      2. Contraindications include maternal fever, preexisting CNS disease, severe hypertension, hypotension, hypovolemia, and coagulopathy.
  4. Psychologic Methods of Pain Relief. Lamaze classes aid in preparation; hypnosis, acupuncture, and biofeedback are also used.

Obstetrics: Vaginal Delivery

  1. Normal Spontaneous Vaginal Delivery.
    1. Cardinal movements (for vertex presentation).
      1. Engagement. Occurs late in pregnancy for primigravida, at the onset of labor for multigravida.
      2. Flexion. Of the neck so that the smallest diameter possible presents. If the neck does not flex, it may actually extend during labor, producing a brow or face presentation.
      3. Descent. Progressive with thinning of the cervix and lower uterine segment. Depends on the force of contractions and on pelvic and presenting part configuration.
      4. Internal rotation. Occurs during descent. Vertex rotates from transverse to either posterior or anterior position to pass the ischial spines.
      5. Extension. Occurs as the head distends the perineum and the occiput passes beneath the symphysis.
      6. External rotation. Occurs after delivery of the head with the head rotating back to a transverse position as the shoulders internally rotate to an anteroposterior position.
    2. Management of vertex delivery.
      1. Preparations for delivery. Should be made when the presenting part begins to distend the perineum, sooner for multigravida. (Local or pudendal anesthesia can be administered at this time.) Episiotomy (if needed) is not performed until delivery is imminent. Episiotomy likely increases the risk of third- and fourth-degree tears.
      2. Delivery of the head.
        1. Controlled so that there is no forceful, sudden expulsion that may produce injury. As the vertex appears beneath the symphysis, the perineum is supported by direct pressure from a draped hand over the coccygeal region (Ritgen’s maneuver). This will protect the perineum and assist in extension of the head as the vertex passes the symphysis.
        2. As the head is delivered, it will rotate to a transverse position, at which time the baby should be checked for the presence of umbilical cord about the neck. If present, it should be gently slipped over the infant’s head (or double clamped and cut if this cannot be done easily).
        3. The mouth and nose should be cleared of secretions with a bulb syringe or DeLee suction trap.
      3. Delivery of the shoulders. Shoulders should be rotated to an AP position in the pelvic outlet as the head externally rotates. Gentle traction downward on the head will assist in bringing the anterior shoulder beneath the symphysis. Gentle elevation of the infant head toward the symphysis will release the posterior shoulder.
      4. Delivery of the body. The rest of the body will generally deliver spontaneously and quickly after delivery of the shoulders. Care must be taken to control the delivery of the body to prevent unnecessary injury.
      5. Immediate care of the infant. Includes double clamping and cutting of the umbilical cord. Do not milk the cord. The clamp closest to the umbilicus should be just distal to the skin or longer if anticipate a need for an umbilical line. A clear airway must be assured and body temperature maintained by drying and wrapping, placing under a radiant heater, or in skin-to-skin contact with mother’s chest.
    3. Forceps delivery: Forceps are generally used to shorten the second stage of labor when in the best interest of the mother or the fetus. A fully dilated cervix and experienced physician are required. Advantages must be weighed against the increased risk of maternal lacerations.
      1. Indications.
        1. Prolonged second stage.
          1. Primigravida with regional anesthesia >3 hours.
          2. Primigravida without regional anesthesia >2 hours.
          3. Multigravida with regional anesthesia >2 hours.
          4. Multigravida without regional anesthesia >1 hour.
        2. Fetal distress.
        3. Maternal exhaustion.
      2. Requirements.
        1. Fetal head engaged and in vertex-face presentation.
        2. Position of head known exactly.
        3. Membranes ruptured.
        4. Cervix fully dilated.
        5. No clinical evidence of cephalopelvic disproportion.
      3. Definitions.
        1. Outlet forceps. The fetal scalp is visible at the introitus. The head is at or on the perineum, and the sagittal suture is in the AP plane or rotated up to 45 degrees.
        2. Low forceps. The leading point of the skull is at least at +2 station.
        3. Midforceps. The leading point of the skull is engaged but is above +2 station. (Midforceps delivery should be attempted only in extreme situations while simultaneously preparing for C-section.)
      4. Selection of forceps.
        1. Simpson. Good for primigravida with prolonged second stage (molded fetal head).
        2. Elliot. Better if multigravida and if less molded fetal head.
        3. Tucker-McLane. Has sliding lock, good for asynclitic fetal head.
        4. Kielland. Has minimal pelvic curve, often used for rotation.
        5. Piper. Used in breech extractions.
    4. Vacuum Extraction A safe, effective alternative to forceps delivery. A term, vertex fetus is required. Delivery should not be one that will require rotation or excessive traction. Prior scalp sampling is a contraindication.
      1. Advantages.
        1. Simpler to apply with fewer mistakes in application.
        2. Less force applied to fetal head.
        3. Less anesthesia necessary (local anesthetic may suffice).
        4. No increase in diameter of presenting head.
        5. Less maternal soft-tissue injury.
        6. Less fetal injury.
        7. Less parental concern.
      2. Disadvantages.
        1. Traction applied only during contractions.
        2. Proper traction necessary to avoid losing vacuum.
        3. Possible longer delivery than with forceps.
        4. Small increase in incidence of cephalohematomas.
      3. Technique.
        1. Ascertain that the cervix is fully dilated and the head is in low or outlet position.
        2. The head is then wiped clean, the labia are spread, and the cup is compressed and inserted. Pressure is applied inward and downward until contact is made with the fetal scalp. The cup should be placed over the posterior fontanelle.
        3. A finger is swept around the cup to make sure no maternal tissue is within the cup. Suction pressure is raised to 100 mm Hg, and the location of the cup is rechecked.
        4. With the onset of a contraction, suction pressure is raised to a range of 380 to 580 mm Hg. (Negative pressure should not exceed 600 mm Hg.) Traction is applied perpendicularly to the cup, in line with the maternal axis.
        5. Should the cup be dislodged, the fetal scalp is to be checked before the cup is reapplied.
        6. When the contraction subsides, the suction pressure is reduced to 100 mm Hg.
        7. As the head crowns, an episiotomy may be cut but likely increases the risk of third- and fourth-degree tears. Traction is then changed to a 45-degree angle upward as the vertex clears the symphysis.
        8. Suction is released and the cup removed after delivery of the fetal head.
        9. The procedure should be discontinued if one fails to achieve extraction after 10 minutes at maximal pressure, extraction is not achieved within 30 minutes of initiation, the cup disengages three times, fetal scalp trauma is sustained, or no progress is made after three pulls.

Obstetrics: Breech Delivery

  1. Overview.
    1. Incidence. 25% of all pregnancies <28 weeks of gestation, 3% to 4% of all pregnancies at or beyond 34 weeks of gestation.
    2. Cause. Low birth weight, placenta previa, uterine and fetal anomalies, contracted pelvis, multiple fetuses all contribute to breech presentations.
  2. Types of Breech.
    1. Frank. Thighs and hips flexed, knees extended. 65% of cases are frank.
    2. Complete. Thighs and hips flexed, one or both knees flexed. 10% of cases.
    3. Incomplete or footling. One or both thighs extended, one or both knees below the buttocks. 25% of cases.
  3. Criteria for Vaginal Delivery of Breech Presentation. (Technique is beyond the scope of this manual.)
    1. Frank breech presentation.
    2. Fetal weight 2500 to 3800 g.
    3. Fetal head flexed.
    4. Gestational age at or beyond 36 weeks.
    5. Adequate maternal pelvis.
    6. No other maternal or fetal indicator for C-section.

Obstetrics: Episiotomy

  1. Overview. A deliberate incision in the perineum used to facilitate vaginal delivery. Stretching of the vaginal tissues manually may prevent the need for episiotomy and minimize the risk of tears.
  2. Midline. Good anatomic results, easy repair, low incidence of postpartum pain or dyspareunia. However, increases the risk of a third or fourth degree laceration compared to patients without an episiotomy.
  3. Mediolateral. Less likely to extend through the sphincter but more likely to cause pain during healing, dyspareunia, or excessive blood loss. Good anatomic results are more difficult to obtain.

Obstetrics: Shoulder Dystocia

  1. Incidence. Directly related to fetal size: >2500 g 0.15%; >4000 g 1.7%; >4500 g 10.0%.
  2. Diagnosis. Suspect shoulder dystocia if there is reason to suspect macrosomia (gestational diabetes, history of large infants, large maternal size, prolonged gestation), or if second stage is prolonged. Consider C-section. In vaginal deliveries, suspect dystocia if the head pulls back against the perineum after delivery, and external rotation is difficult.
  3. Management.
    1. Ensure adequate maternal anesthesia and cut a very generous episiotomy.
    2. Attempt McRobert’s maneuver. The mother’s thighs are hyperflexed, bringing her feet "to her ears." Have an assistant apply suprapubic pressure. This causes the shoulder to move under the symphysis pubis. Attempt delivery with gentle downward traction.
    3. Attempt the Wood’s screw maneuver. Gently rotate the posterior shoulder by pushing on the posterior scapula until the shoulder passes under the symphysis and can be delivered as the anterior shoulder.
    4. If this is unsuccessful, try delivering the posterior arm first and then rotating the anterior shoulder into the oblique position for delivery.
    5. If all else fails, one may attempt deliberate fracture of the clavicle of the impacted shoulder. The thumb and forefinger are used to push the clavicle outward to avoid a pneumothorax. Although the fracture will heal, damage to cervical nerve roots may occur and cause permanent sequelae.

Obstetrics: Cesarean Section

  1. Indications.
    1. Maternal and fetal. Cephalopelvic disproportion, failed induction or progression of labor, abnormal uterine contraction pattern.
    2. Maternal.
      1. Maternal diseases. Eclampsia or preeclampsia with non-inducible cervix, diabetes mellitus (if macrosomic infant precludes vaginal delivery), cardiac disease, cervical cancer, active herpes genitalis. One double-blind clinical trial showed that acyclovir suppression (400 mg PO TID) given after 36 weeks of gestation significantly reduces the need for cesarean section by preventing a herpetic outbreak at term.
      2. Previous uterine surgery. Classic cesarean section, previous uterine rupture, full-thickness myomectomy. If there is any question about the type of incision made during a previous cesarean section, the operative report for that delivery must be obtained so that incisional type can be known with certainty.
      3. Obstruction to the birth canal. Fibroids, ovarian tumors.
    3. Fetal. Fetal distress, cord prolapse, fetal malpresentations.
    4. Placental. Placenta previa (unless marginal) and abruptio placentae.
  2. Risks.
    1. Maternal. Infection, hemorrhage, injury to urinary tract, adverse reactions to anesthesia, prolonged recovery.
    2. Fetal. Depends on gestational age and indications for C-section. Less birth trauma, though injury can be sustained during operative delivery. May have increased incidence of respiratory distress syndrome.
  3. Antibiotic Prophylaxis. Reduced incidence of endometritis, wound infection, urinary tract infection and fever post-op with single dose of antibiotic (e.g., ceftriaxone, cefotetan) prior to caesarean delivery.

Obstetrics: Postpartum Hemorrhage

  1. Definition. Postpartum hemorrhage is most often defined as a blood loss greater than 500 ml in the first 24 hours after delivery. However, blood loss after spontaneous vaginal delivery is frequently up to 600 ml and between 1 and 1.5 liters after instrumental or operative delivery. Therefore, clinical experience is necessary to determine when bleeding is occurring too rapidly, at the wrong time or is unresponsive to appropriate treatment. Blood loss will be less well tolerated if the patient has not had the normal expansion of blood volume during pregnancy, as in cases of preeclampsia.
  2. Risk Factors. Multiparity (>5 babies), previous postpartum hemorrhage, manual removal of the placenta, placental abruption or placenta previa, polyhydramnios, prolonged labor, precipitant labor, difficult forceps delivery, prolonged oxytocin administration, breech extraction.
  3. Etiology. Uterine atony accounts for most cases. Other causes include retained placenta, cervical or vaginal tear, and coagulopathy.
  4. Physical Exam.
    1. Vital signs. BP and pulse abnormalities are very late signs of bleeding due to hemodynamics of pregnancy and usual young age of patient.
    2. Uterus should be palpated for evidence of atony, tenderness, or lack of involution.
    3. Vaginal exam may reveal evidence of laceration (generally bright red blood) or atony (darker blood). Bimanual exam may reveal mass (suggestive of broad ligament or paravaginal hematoma).
    4. Hematocrit is helpful only in comparison to the value before delivery. It will not adequately reflect acute blood loss.
  5. Management.
    1. Reliable IV access must be obtained with 2 large-bore IVs. Monitor vital signs and maintain circulatory status with fluids. If the patient shows evidence of symptomatic hypovolemia, blood should be sent for type and cross. Coagulation profile should also be obtained.
    2. Review clinical course for probable cause (see predisposing factors listed above).
    3. Perform bimanual examination in recovery area or delivery room.
    4. Managing specific causes of postpartum hemorrhage.
      1. Uterine atony:
        1. Uterine massage.
        2. Oxytocin IM (10-20 U)/IV (40 U/L at 250 ml/hr) if no contraindications.
        3. Methergine IM (0.2 mg ) contraindicated if hypertension, pre-eclampsia hypersensitivity.
        4. Prostaglandin F2 (Hemabate) IM or intramyometrially 0.25 mg Q15 minutes up to 8 doses. Contraindicated if active cardiac, renal, pulmonary, or hepatic disease.
      2. Retained placenta or invasive placenta. Manual removal of placenta, identify cleavage plain with intrauterine hand, advance fingertips to separate. If can not identify cleavage- probably invasive placenta and requires surgery.
      3. Trauma. Identify laceration or hematoma and repair. Consider uterine inversion and uterine rupture.
      4. If cause is not identified or fails to respond to the above measures, notify obstetric physicians, anesthesia, and operating room personnel of potential need for surgical intervention. Inform patient of the problem and what measures are being taken to correct it. Get an appreciation of her desires regarding further childbearing and hysterectomy.
      5. If uterine bleeding persists, surgery must be considered. Packing or balloon tamponade (e.g., 24 French Foley with 70-80 cc water) is a temporary measure and is rarely effective. Surgical alternatives include uterine artery and hypogastric artery ligation. Hysterectomy is the treatment of last resort when the patient desires future fertility but may be preferred if sterility is desired.

Obstetrics: Postpartum Care

  1. Examples of Orders after Routine Vaginal Delivery.
    1. Immediately postpartum:
      1. Pitocin 10 units IM.
      2. Bed rest, and vitals Q15 min for 1 hour postpartum.
      3. Ice pack to perineum immediately postpartum PRN.
    2. Thereafter:
      1. Ambulate as soon as possible.
      2. Diet. General or other.
      3. Vital signs. Q4h.
      4. Tucks to perineum PRN.
      5. Sitz baths TID and HS PRN.
      6. IV (if present). Discontinue when vital signs are stable and uterine bleeding is normal.
      7. Bladder catheterization if unable to void in 6 to 8 hours.
      8. Breast binder if not nursing.
      9. CBC postpartum day 2.
      10. Administer Rho(D) immunoglobulin if indicated.
      11. Medications.
        1. Vitamins. Continue prenatal vitamins; additional FeSO4 if anemic.
        2. Pain. Acetaminophen 650 mg PO Q4-6h PRN or ibuprofen 400 to 600 mg PO Q4-6h for cramping pain. Narcotics as needed (but be careful if breast feeding).
        3. Bowels. Docusate sodium 100 mg PO BID; milk of magnesia 30 ml PO QD PRN; bisacodyl 10 mg PO or PR PRN.
  2. Hospital Care.
    1. Physical examination.
      1. Monitor uterine changes. The fundus should be firm and at or below the umbilicus. Gradual involution occurs over the next 6 weeks.
      2. Lochia (uterine drainage) is initially red or bloody, gradually becoming serosanguinous. By 2 to 3 weeks it should be white. Tampons are contraindicated.
      3. Breasts are examined for signs of infection and presence of milk. Colostrum is present initially. Milk production should occur by the third to fifth day in primiparas, sooner in multiparas. Breast feeding should not be allowed for greater than 15 minutes on each side per feeding initially to help prevent soreness.
      4. Legs should be examined for evidence of thrombophlebitis.
  3. Parent Education.
    1. Newborn care.
    2. Breast feeding and prevention of lactation or engorgement if applicable.
  4. Discharge.
    1. Discharge instructions.
      1. Rubella vaccination, if indicated, before discharge.
      2. Instruct regarding signs of puerperal infection, postpartum hemorrhage, and mastitis.
      3. Counsel on avoidance of intercourse and tampons for 4 weeks.
      4. Contraception counseling. OCPs can be started during week 4, if desired. Low-dose or progestin-only pills and Depo-Provera have less influence on lactation.
      5. Nutrition. Especially if breast feeding.
      6. Medications. Vitamins, iron, stool softener, when appropriate. Counsel on medications to avoid during breast feeding.
      7. Discuss need for rest, possible stresses that can occur with new infant at home, possibility of postpartum depression.
  5. Follow-up Exam.
    1. Postpartum check at 4 to 6 weeks.
    2. Newborn checkup typically at 1 to 2 weeks.

Obstetrics: Puerperal Fever

  1. Definition. Temperature >38.4° C in first 24 hours or >38.0° C for 2 consecutive days in the 9 days following delivery.
  2. Differential Diagnosis.
    1. Endometritis.
      1. Etiology. Polymicrobial with a mixture of aerobic and anaerobic organisms. In particular, high fever within the first 25 hours after delivery may be caused by gram-negative sepsis, group B streptococcal disease, clostridial sepsis, or toxic shock syndrome. Those 2 days to 6 weeks postpartum may be secondary to Chlamydia.
      2. Risk factors. C-section (20 times greater than vaginal delivery), chorioamnionitis, prolonged rupture of membranes or premature labor, multiple vaginal exams, retained products, manual exploration of the uterus, low socioeconomic status.
      3. Treatment.
        1. Cultures of the cervix and blood may help identify the causative organism, but treatment is often started empirically. If Chlamydia is isolated or suspected based on late presentation, add doxycycline or azithromycin to the regimen.
        2. There is no consensus on the safest and most effective antibiotic regimens, only that it must have a broad spectrum. Antibiotics are usually continued for 4 or 5 days and for 24 to 48 hours after defervescence.
        3. "Gold standard" = Gentamicin (2 mg/kg IV loading dose, followed by 1.5 mg/kg IV Q8h) + Clindamycin (900 mg IV Q8h).
        4. Newer regimens (second- or third-generation cephalosporins, semisynthetic penicillins).
          1. Cefoxitin 1 to 2 g IV Q6-8h.
          2. Ampicillin/sulbactam 1.5-3 g IV Q6h.
        5. If no response (maximum temperature not dropping within 48 hours of initiation of therapy), start triple-agent therapy: ampicillin and gentamicin and clindamycin, or ampicillin/sulbactam or cefotoxin + ampicillin.
    2. Pelvic abscess. Suspect if patient develops a pelvic mass or has persistent fever and pain despite therapy for aerobic bacteria. Frequently develops 5 or more days after delivery. Must add therapy for anaerobic bacteria and consider surgical or percutaneous drainage.
    3. Septic pelvic thrombophlebitis. Symptoms include spiking fevers with or without pain despite antibiotic therapy. The patient may have a tender palpable mass. May have a diagnostic response with improvement of symptoms after beginning intravenous heparin and antibiotics. CT and MRI have been used to diagnose this illness.
    4. Wound infection. Presentation includes fever, a tender, erythematous, or fluctuant incision, drainage of pus or blood. Usually occurs after the fifth postoperative day. Risk factors include having an intrapartum cesarean section; emergent abdominal delivery, use of electrocautery, placement of open drains, obesity, and diabetes.
    5. Pulmonary atelectasis. See Chapter 15 for a discussion of pulmonary atelectasis and fever.
    6. Deep vein thrombosis. Symptoms include fever and lower extremity pain, swelling, and pallor. Traumatic delivery, cesarean section, delay in the resumption of ambulation, and varicose veins all increase likelihood for DVT formation. See Chapter 4.
    7. Pyelonephritis. Often accompanied by fever, malaise, flank pain, costovertebral angle tenderness, and pyuria. Risk factors include occult bacteriuria, bladder trauma, and Foley catheterization.
    8. Mastitis. Suggested by fever and swollen, tender breast. Typically occurs 3 to 4 weeks after delivery. Breast feeding and contact with a carrier of Staphylococcus aureus are the two prime risk factors.